Inverse Laplace Transfrom - Taylor Series/Asymptotic Series?

Click For Summary
The discussion centers on finding the inverse Laplace transform of the function W(x,s) = (1/s)*(sinh(x*s^0.5))/(sinh(s^0.5)). The solution provided is w(x,t) = x + Σ((-1)^n/n) * e^(-t*(n*pi)^2) * sin(n*pi*x), summing from n=1 to ∞. The user attempts to simplify W(x,s) using the Taylor series for sinh, leading to an expression that complicates their understanding of the asymptotic series. There is a mention of using a branch cut and the Bromwich inversion formula for the inverse transform, indicating the complexity of the problem. Overall, the user seeks clarification on integrating these concepts into their solution for the non-homogeneous PDE.
chrissimpson
Messages
11
Reaction score
0

Homework Statement



W(x,s)=(1/s)*(sinh(x*s^0.5))/(sinh(s^0.5))

Find the inverse laplace transform of W(x,s), i.e. find w(x,t).

Answer:

w(x,t)= x + Ʃ ((-1)^n)/n) * e^(-t*(n*pi)^2) * sin(n*pi*x)

summing between from n=1 to ∞


Homework Equations



An asymptotic series..?!

The Attempt at a Solution



Taking the Taylor series for sinh 'simplifies' the expression to:

W(x,s)=(1/s)*(x + x^3 + x^5 +x^7 ...)

You can write this as:

W(x,s)=(x/s)+(1/s)*(x^3 + x^5 + x^7...)

W(x,s)= (x/s) + Ʃ(x^2n+1)/s

summing between n=1 and ∞

This is where I think I begin to run into trouble (unless I already have!) as I think I would now form:

w(x,t)= x + Ʃ(x^2n+1)


There is some mention in a similar question that an asymptotic series was used, but I can't work out how to make this fit in with the question!


Any help would be really appreciated!

Chris
 
Physics news on Phys.org
I got involved with things like this when I was looking at plasma physics, you want to use a branch cut in you inverse transform and usae the Bromwich inversion formula.
 
Cheers! I'll have a look into that. Any idea how painfully difficult this will be - I'm coming towards the (current) limit of my mathematical capabilities with this non-homogeneous PDE stuff!
 
Question: A clock's minute hand has length 4 and its hour hand has length 3. What is the distance between the tips at the moment when it is increasing most rapidly?(Putnam Exam Question) Answer: Making assumption that both the hands moves at constant angular velocities, the answer is ## \sqrt{7} .## But don't you think this assumption is somewhat doubtful and wrong?

Similar threads

  • · Replies 1 ·
Replies
1
Views
2K
Replies
2
Views
2K
  • · Replies 7 ·
Replies
7
Views
2K
  • · Replies 10 ·
Replies
10
Views
2K
Replies
2
Views
2K
  • · Replies 9 ·
Replies
9
Views
3K
Replies
9
Views
2K
  • · Replies 27 ·
Replies
27
Views
3K
  • · Replies 3 ·
Replies
3
Views
2K
  • · Replies 5 ·
Replies
5
Views
3K